LSAT and Law School Admissions Forum

Get expert LSAT preparation and law school admissions advice from PowerScore Test Preparation.

 Smdickso
  • Posts: 2
  • Joined: Sep 14, 2015
|
#19833
Thank you so much, you were a great help & explained it perfectly. You are right, I don't think I will miss it again! :)
 mamalit1
  • Posts: 6
  • Joined: Jan 13, 2016
|
#22058
Hi, Please i need help to understand something.December 2000, game 5 question about the birds in the forest, i made all the necessary inferences and also made the inference between M and S which is wrong and i don't understand why.
if J is in then H is in and G is not
if M is in then H is in i inferred that M and J can both be together but that inference is not in the book i don't know why also
J is in then S is not, so then I thought M and S shouldn't be in the forest together and likewise H and S shouldn't be in together that apparently that is a wrong inference. why is that?
I got all the other linkages right though
Thanks
User avatar
 Dave Killoran
PowerScore Staff
  • PowerScore Staff
  • Posts: 5852
  • Joined: Mar 25, 2011
|
#22060
Hi Mama,

Thanks for the questions! This is a tricky game (and that's one reason I included it in the LSAT Logic Games Bible), and thus it's come up a lot on this forum. So, I'm going to start by referring you to the posts I wrote about this game on page 1 of this thread:

lsat/viewtopic.php?f=7&t=7218&p=18409

In those posts, focus more on what I'm saying than the original questions; in the process of answering each question I explain the rules that make this game tough.

With inferences, remember that they typically revolve around what must be true or what cannot be true. Thus, an inference that two birds could be in the forest together isn't something you'd see me note very often, simply because if you were writing down what could be true, you could be at it for a very long time. So you will instead see absolutes noted, not possibilities.

If you don't mind, check out the two posts above, and if you have questions after that, let me know and then I'll go into further detail. Thanks!
 yrivers
  • Posts: 68
  • Joined: Mar 15, 2017
|
#34104
Hi,

Thanks for the previous explanations on this! I have 2 questions, specifically:

8) For #8 in the problem set, it's stated that "jays are not in the forest". Does this mean jays are the one and only ones in the forest? Meaning, can I bring in the rules "if no G then no W" and "if no H then no J and no M"? Or does it need to be stated in the problem that jays are the one and only birds? I have a hard time differentiating when to "infer" and not to bring this sort of inference in solving the problem.

9) In a condition undefined problem like this, what's the most efficient, fastest way to approach maximum/minimum problems?

Thank you,
Yaesul
 Robert Carroll
PowerScore Staff
  • PowerScore Staff
  • Posts: 1787
  • Joined: Dec 06, 2013
|
#34115
Yaesul,

Be careful! The local condition in question #8 says that jays are not in the forest. Thus, jays cannot be in the forest. Further, if a hypothetical local question said "X is in Y," that doesn't imply that only X is in Y. Nothing should be assumed to list completely all the things that are in unless that conditions says it's a complete list.

For question #9, you should have a substantial number of inferences and rule representations already written before you began the questions. When this asks you what the maximum number is, you first look at variables, if any, that are in multiple Double-Not Arrows. Here, if W is in, H, J, and M cannot be in. Thus, if W is in, the maximum is 3. Can we do better than that? If we can, it has to be by leaving W out. So leave W out. We know that answer choice (E) isn't the answer now (otherwise, W is in, but that excludes 3 variables, so it can't be 6 in!). Now we look at the 5 variables left and see if any two are related by Double-Not Arrows - because they are incompatible, those two couldn't both be in, meaning 5 wouldn't actually work. G conflicts with H, J, and M, so exclude G. You're left with H, J, M, and S. Including all 4 of these doesn't violate a rule, whereas trying to get 5 or 6 will cause problems with Double-Not Arrows, so we know answer choice (D) is wrong and answer choice (C) is the correct answer.

Robert Carroll
 anabelgvr
  • Posts: 2
  • Joined: Jul 07, 2017
|
#36928
Hello,

thank you for the explanation but I'm still having trouble understanding one of the questions in this game. My question is regarding question # 11. I don't understand what happens to W. Why is it a 'free" variable. Why can it choose to be in or out? How do I know when a variable can no longer be bound to the rule?

Thank you
User avatar
 Jonathan Evans
PowerScore Staff
  • PowerScore Staff
  • Posts: 726
  • Joined: Jun 09, 2016
|
#37133
Hi, Anabelgvr,

Good question, and in fact you have somewhat expressed exactly what it means to be a "free variable":
  • W could be in the forest, but it is not necessarily in the forest.
W is "free" in that it could be could be in or it could be out; G in the forest does not imply one location or the other for W. Follow the chain conditional:
  • G :arrow: H :arrow: J & M :arrow: S
How many of the birds have we accounted for here? We have accounted for five of the six. G started us off by being in the forest. That was sufficient to know what happens to four other birds. However, there's one bird not on the receiving end of anything in this chain: W. W could be either in or out of the forest.

Thus, the overall breakdown looks like this:
  • Selection: G S (W/ _ )
    Not Selected: H J M ( _ /W)
Does this help?
 moalkhaf
  • Posts: 10
  • Joined: Aug 30, 2017
|
#40083
Good afternoon. I was wondering if someone could clarify why the following diagram for this game is incorrect.
H<-|-> G

J
and -----> H
M
J---->H
M---->H

W---->G
J<-|-> S

Inferences
J<-|->G
M<-|->G
W<--|-->H
S<--|-->J--->h
S<--|-->H
User avatar
 Dave Killoran
PowerScore Staff
  • PowerScore Staff
  • Posts: 5852
  • Joined: Mar 25, 2011
|
#40087
Hi M,

thanks for the question! Since you appear to have the LGB, check the explanation for this game. In your diagram, you made an error with the representation of the last rule. You diagrammed it as: J :dblline: S, when in fact the correct super-diagram is J :dblline: S.

The negative being on the sufficient condition (vs the necessary, which is what creates your diagram) creates an entirely different diagram than the standard double-arrow, and I talk about that in detail in the game explanation (and earlier in the book). I also talk about it here: The Most Dangerous Conditional Rule on the LSAT. I recommend reading that article since it should clarify the difference between what is possible in each representation.

Discussion of this game also come up frequently on this Forum, and you might find those helpful as well! Here are a few:


Interestingly, that one rule representation difference can really cause problems when doing this game, so it's good that you came across this now. If you have any questions on that after reading through those, just let me know. Thanks!
 moalkhaf
  • Posts: 10
  • Joined: Aug 30, 2017
|
#40112
Good afternoon. I appreciate your response. That clarifies the game.

Get the most out of your LSAT Prep Plus subscription.

Analyze and track your performance with our Testing and Analytics Package.